2022 AMC 12A Problems/Problem 12

Revision as of 11:20, 12 November 2022 by Oxymoronic15 (talk | contribs) (Created page with "==Problem== Let <math>M</math> be the midpoint of <math>AB</math> in regular tetrahedron <math>ABCD</math>. What is <math>\cos(\angle CMD)</math>? <math>\textbf{(A) } \frac1...")
(diff) ← Older revision | Latest revision (diff) | Newer revision → (diff)

Problem

Let $M$ be the midpoint of $AB$ in regular tetrahedron $ABCD$. What is $\cos(\angle CMD)$?

$\textbf{(A) } \frac14 \qquad \textbf{(B) } \frac13 \qquad \textbf{(C) } \frac25 \qquad \textbf{(D) } \frac12 \qquad \textbf{(E) } \frac{\sqrt{3}}{2}$